DOWNLOAD IJMB PAST QUESTIONS WITH DETAILED SOLUTIONS HERE
SECTION A
Question 1
Given that $$\sec \theta = \frac{25}{24} $$, compute the value of $$\cot \theta + \cos \theta $$.
Solution: First, let’s find $$\cos \theta $$ using the relationship $$\sec \theta = \frac{1}{\cos \theta} $$:
$$\sec \theta = \frac{25}{24} $$
$$\cos \theta = \frac{1}{\sec \theta} = \frac{24}{25} $$
Now, to find $$\cot \theta $$, we need $$\sin \theta $$. Using the Pythagorean identity: $$\sin^2 \theta + \cos^2 \theta = 1 $$
$$\sin^2 \theta = 1 – \cos^2 \theta = 1 – \left(\frac{24}{25}\right)^2 = 1 – \frac{576}{625} = \frac{625-576}{625} = \frac{49}{625} $$
$$\sin \theta = \frac{7}{25} $$ (taking the positive value since $$\sec \theta $$ is positive, suggesting we’re in the first quadrant)
Now we can calculate $$\cot \theta $$: $$\cot \theta = \frac{\cos \theta}{\sin \theta} = \frac{24/25}{7/25} = \frac{24}{7} $$
Finally, we compute $$\cot \theta + \cos \theta $$: $$\cot \theta + \cos \theta = \frac{24}{7} + \frac{24}{25} = \frac{24 \cdot 25 + 24 \cdot 7}{7 \cdot 25} = \frac{24(25+7)}{175} = \frac{24 \cdot 32}{175} = \frac{768}{175} $$
Therefore, $$\cot \theta + \cos \theta = \frac{768}{175} $$
Question 2
If $$\alpha $$ and $$\beta $$ are the roots of the equation $$x^2 – 5x + 6 = 0 $$, find $$\alpha – \beta $$.
Solution: From Vieta’s formulas, for a quadratic equation $$ax^2 + bx + c = 0 $$, the sum and product of roots are given by:
- $$\alpha + \beta = -\frac{b}{a} $$
- $$\alpha \cdot \beta = \frac{c}{a} $$
For $$x^2 – 5x + 6 = 0 $$, we have $$a = 1 $$, $$b = -5 $$, and $$c = 6 $$.
Therefore:
- $$\alpha + \beta = -\frac{-5}{1} = 5 $$
- $$\alpha \cdot \beta = \frac{6}{1} = 6 $$
To find $$\alpha – \beta $$, we use the relationship: $$(\alpha – \beta)^2 = (\alpha + \beta)^2 – 4\alpha\beta $$
$$(\alpha – \beta)^2 = 5^2 – 4 \cdot 6 = 25 – 24 = 1 $$
$$\alpha – \beta = \pm 1 $$
Since we’re just asked for $$\alpha – \beta $$ without specifying which is which, either value is valid. Conventionally, if we consider $$\alpha > \beta $$, then $$\alpha – \beta = 1 $$.
Question 3
Express $$\left(\frac{2z+1}{z-1}\right)^2 $$ in the form $$a + ib $$.
Solution: Let’s denote the complex expression as $$\left(\frac{2z+1}{z-1}\right)^2 $$.
We need to simplify this expression. First, let’s work with $$\frac{2z+1}{z-1} $$:
$$\frac{2z+1}{z-1} = \frac{2z-2+2+1}{z-1} = \frac{2(z-1)+3}{z-1} = 2 + \frac{3}{z-1} $$
Now we need to square this result: $$\left(2 + \frac{3}{z-1}\right)^2 = 4 + \frac{12}{z-1} + \frac{9}{(z-1)^2} $$
This is a general form. To express in the form $$a + ib $$, we would need specific values of $$z $$. However, since we’re not given specific values, we can leave it in this algebraic form.
Note: The question likely intended to substitute a specific complex value for $$z $$, but it appears to be missing from the image. Without a specific value of $$z $$, we cannot express the final answer in the exact form $$a + ib $$.
Question 4
If the 16th term of an AP is 3 times the 4th term, prove that the 23rd term is 5 times the 3rd term.
Solution: Let’s denote the first term of the AP by $$a $$ and the common difference by $$d $$.
The $$n $$-th term of an AP is given by $$T_n = a + (n-1)d $$.
Given information: $$T_{16} = 3T_4 $$
Let’s substitute: $$a + (16-1)d = 3[a + (4-1)d] $$ $$a + 15d = 3(a + 3d) $$ $$a + 15d = 3a + 9d $$ $$a – 3a = 9d – 15d $$ $$-2a = -6d $$ $$a = 3d $$
Now, we need to prove that $$T_{23} = 5T_3 $$:
$$T_{23} = a + (23-1)d = a + 22d = 3d + 22d = 25d $$ $$T_3 = a + (3-1)d = a + 2d = 3d + 2d = 5d $$
Therefore: $$T_{23} = 25d = 5 \cdot 5d = 5T_3 $$
Thus, the 23rd term is 5 times the 3rd term.
Question 5
Solve the equation $$1^x = 3 $$.
Solution: We need to solve $$1^x = 3 $$.
Note that $$1 $$ raised to any power is always equal to $$1 $$: $$1^x = 1 $$ for any value of $$x $$.
However, our equation states $$1^x = 3 $$, which is impossible since $$1 $$ raised to any power cannot equal $$3 $$.
Therefore, the equation $$1^x = 3 $$ has no solution.
SECTION B: ALGEBRA
Question 6(a)
Given that $$x – 1 $$ is a factor of the polynomial $$px^2 + qx + r $$, and when the polynomial is divided by $$x + 1 $$ and $$x – 2 $$ leaves remainders of $$2 $$ and $$8 $$, respectively. Calculate the values of $$p $$, $$q $$ and $$r $$.
Solution: If $$x – 1 $$ is a factor of $$px^2 + qx + r $$, then $$p(1)^2 + q(1) + r = 0 $$, which simplifies to: $$p + q + r = 0 $$ … (1)
When $$px^2 + qx + r $$ is divided by $$x + 1 $$, the remainder is $$2 $$. By the remainder theorem, if $$x = -1 $$, then $$p(-1)^2 + q(-1) + r = 2 $$: $$p – q + r = 2 $$ … (2)
When $$px^2 + qx + r $$ is divided by $$x – 2 $$, the remainder is $$8 $$. By the remainder theorem, if $$x = 2 $$, then $$p(2)^2 + q(2) + r = 8 $$: $$4p + 2q + r = 8 $$ … (3)
From equations (1) and (2): $$p + q + r = 0 $$ … (1) $$p – q + r = 2 $$ … (2)
Subtracting (2) from (1): $$2q = -2 $$ $$q = -1 $$
Substituting $$q = -1 $$ into equation (1): $$p + (-1) + r = 0 $$ $$p + r = 1 $$ … (4)
Now from equations (3) and (4): $$4p + 2(-1) + r = 8 $$ $$4p – 2 + r = 8 $$ $$4p + r = 10 $$ … (5)
Subtracting (4) from (5): $$3p = 9 $$ $$p = 3 $$
Substituting $$p = 3 $$ into equation (4): $$3 + r = 1 $$ $$r = -2 $$
Therefore: $$p = 3 $$, $$q = -1 $$, and $$r = -2 $$
Question 6(b)
Obtain the first four terms of the binomial expansion of $$(x – y)^{10} $$ and use it to estimate the value of $$(0.99)^{10} $$, correct to 3 decimal points.
Solution: Using the binomial theorem:
$$(x – y)^{10} = \sum_{k=0}^{10} \binom{10}{k} x^{10-k} (-y)^k $$
The first four terms are: $$(x – y)^{10} = \binom{10}{0} x^{10} (-y)^0 + \binom{10}{1} x^9 (-y)^1 + \binom{10}{2} x^8 (-y)^2 + \binom{10}{3} x^7 (-y)^3 + … $$
$$= x^{10} – 10x^9y + 45x^8y^2 – 120x^7y^3 + … $$
To estimate $$(0.99)^{10} $$, we can rewrite it as $$(1 – 0.01)^{10} $$.
Setting $$x = 1 $$ and $$y = 0.01 $$ in our expansion:
$$(0.99)^{10} = (1 – 0.01)^{10} = 1^{10} – 10 \cdot 1^9 \cdot 0.01 + 45 \cdot 1^8 \cdot (0.01)^2 – 120 \cdot 1^7 \cdot (0.01)^3 + … $$
$$= 1 – 0.1 + 0.0045 – 0.00012 + … $$
$$= 0.90438… \approx 0.904 $$ (to 3 decimal places)
Question 7(a)
Prove that $$2^{4n} – 1 $$ is always divisible by 5.
Solution: We can prove this by showing that $$2^{4n} \equiv 1 \pmod{5} $$ for all positive integers $$n $$.
Let’s examine the powers of 2 modulo 5: $$2^1 \equiv 2 \pmod{5} $$ $$2^2 \equiv 4 \pmod{5} $$ $$2^3 \equiv 8 \equiv 3 \pmod{5} $$ $$2^4 \equiv 16 \equiv 1 \pmod{5} $$
We observe that $$2^4 \equiv 1 \pmod{5} $$, which means the pattern repeats every 4 powers.
For any positive integer $$n $$: $$2^{4n} = (2^4)^n \equiv 1^n \equiv 1 \pmod{5} $$
Therefore: $$2^{4n} – 1 \equiv 1 – 1 \equiv 0 \pmod{5} $$
This proves that $$2^{4n} – 1 $$ is always divisible by 5 for all positive integers $$n $$.
Question 7(b)
Solve the following simultaneous linear equations using Cramer’s rule: $$x + 2y – 2z = 1 $$ $$x – 3y + 8z = 6 $$ $$2x – 3y – z = -2 $$
Solution: Cramer’s rule states that for a system of equations $$AX = B $$, each variable $$x_i $$ can be calculated as $$x_i = \frac{|A_i|}{|A|} $$ where $$|A| $$ is the determinant of coefficient matrix, and $$|A_i| $$ is the determinant of matrix $$A $$ with its $$i $$-th column replaced by the constant column $$B $$.
For our system, we have: $$A = \begin{pmatrix} 1 & 2 & -2 \ 1 & -3 & 8 \ 2 & -3 & -1 \end{pmatrix} $$
$$B = \begin{pmatrix} 1 \ 6 \ -2 \end{pmatrix} $$
First, calculate $$|A| $$: $$|A| = 1 \cdot \begin{vmatrix} -3 & 8 \ -3 & -1 \end{vmatrix} – 2 \cdot \begin{vmatrix} 1 & 8 \ 2 & -1 \end{vmatrix} + (-2) \cdot \begin{vmatrix} 1 & -3 \ 2 & -3 \end{vmatrix} $$
$$= 1 \cdot ((-3)(-1) – 8(-3)) – 2 \cdot (1(-1) – 8(2)) + (-2) \cdot (1(-3) – (-3)(2)) $$
$$= 1 \cdot (3 + 24) – 2 \cdot (-1 – 16) + (-2) \cdot (-3 – (-6)) $$
$$= 1 \cdot 27 – 2 \cdot (-17) + (-2) \cdot 3 $$
$$= 27 + 34 – 6 = 55 $$
Next, calculate $$|A_x| $$ (replace first column with constants): $$|A_x| = \begin{vmatrix} 1 & 2 & -2 \ 6 & -3 & 8 \ -2 & -3 & -1 \end{vmatrix} $$
$$= 1 \cdot \begin{vmatrix} -3 & 8 \ -3 & -1 \end{vmatrix} – 2 \cdot \begin{vmatrix} 6 & 8 \ -2 & -1 \end{vmatrix} + (-2) \cdot \begin{vmatrix} 6 & -3 \ -2 & -3 \end{vmatrix} $$
$$= 1 \cdot (3 + 24) – 2 \cdot (-6 – (-16)) + (-2) \cdot (-18 – 6) $$
$$= 27 – 2 \cdot 10 + (-2) \cdot (-24) $$
$$= 27 – 20 + 48 = 55 $$
Calculate $$|A_y| $$ (replace second column with constants): $$|A_y| = \begin{vmatrix} 1 & 1 & -2 \ 1 & 6 & 8 \ 2 & -2 & -1 \end{vmatrix} $$
$$= 1 \cdot \begin{vmatrix} 6 & 8 \ -2 & -1 \end{vmatrix} – 1 \cdot \begin{vmatrix} 1 & 8 \ 2 & -1 \end{vmatrix} + (-2) \cdot \begin{vmatrix} 1 & 6 \ 2 & -2 \end{vmatrix} $$
$$= 1 \cdot (-6 – (-16)) – 1 \cdot (-1 – 16) + (-2) \cdot (-2 – 12) $$
$$= 1 \cdot 10 – 1 \cdot (-17) + (-2) \cdot (-14) $$
$$= 10 + 17 + 28 = 55 $$
Calculate $$|A_z| $$ (replace third column with constants): $$|A_z| = \begin{vmatrix} 1 & 2 & 1 \ 1 & -3 & 6 \ 2 & -3 & -2 \end{vmatrix} $$
$$= 1 \cdot \begin{vmatrix} -3 & 6 \ -3 & -2 \end{vmatrix} – 2 \cdot \begin{vmatrix} 1 & 6 \ 2 & -2 \end{vmatrix} + 1 \cdot \begin{vmatrix} 1 & -3 \ 2 & -3 \end{vmatrix} $$
$$= 1 \cdot (6 – (-18)) – 2 \cdot (-2 – 12) + 1 \cdot (-3 – (-6)) $$
$$= 1 \cdot 24 – 2 \cdot (-14) + 1 \cdot 3 $$
$$= 24 + 28 + 3 = 55 $$
Using Cramer’s rule: $$x = \frac{|A_x|}{|A|} = \frac{55}{55} = 1 $$ $$y = \frac{|A_y|}{|A|} = \frac{55}{55} = 1 $$ $$z = \frac{|A_z|}{|A|} = \frac{55}{55} = 1 $$
Therefore, the solution is $$x = 1 $$, $$y = 1 $$, $$z = 1 $$.
Question 8(a)
Solve the equation $$\log_5 (x^2 + 9) – \log_5 (x + 1) = 1 $$
Solution: Starting with the equation $$\log_5 (x^2 + 9) – \log_5 (x + 1) = 1 $$
Using the logarithm property $$\log_a(m) – \log_a(n) = \log_a(\frac{m}{n}) $$:
$$\log_5 \left(\frac{x^2 + 9}{x + 1}\right) = 1 $$
Since $$\log_5(5) = 1 $$, we can rewrite:
$$\frac{x^2 + 9}{x + 1} = 5 $$
Multiply both sides by $$(x + 1) $$: $$x^2 + 9 = 5(x + 1) $$ $$x^2 + 9 = 5x + 5 $$ $$x^2 – 5x + 4 = 0 $$
Using the quadratic formula: $$x = \frac{5 \pm \sqrt{25 – 16}}{2} = \frac{5 \pm 3}{2} $$
$$x = 4 $$ or $$x = 1 $$
We need to check if these solutions satisfy the original equation by ensuring the expressions inside the logarithms are positive:
For $$x = 4 $$: $$x^2 + 9 = 16 + 9 = 25 > 0 $$ ✓ $$x + 1 = 4 + 1 = 5 > 0 $$ ✓
For $$x = 1 $$: $$x^2 + 9 = 1 + 9 = 10 > 0 $$ ✓ $$x + 1 = 1 + 1 = 2 > 0 $$ ✓
Both solutions are valid: $$x = 4 $$ or $$x = 1 $$
Question 8(b)
Solve the equation $$49^x – 42^x = 36^x $$.
Solution: Starting with $$49^x – 42^x = 36^x $$
We can rewrite $$49 = 7^2 $$, $$42 = 6 \times 7 $$, and $$36 = 6^2 $$: $$(7^2)^x – (6 \times 7)^x = (6^2)^x $$ $$7^{2x} – (6^x \times 7^x) = 6^{2x} $$
Factoring out $$7^x $$: $$7^{2x} – 7^x \cdot 6^x = 6^{2x} $$ $$7^x(7^x – 6^x) = 6^{2x} $$
Dividing both sides by $$6^x $$: $$\frac{7^x(7^x – 6^x)}{6^x} = 6^x $$ $$\frac{7^x \cdot 7^x}{6^x} – \frac{7^x \cdot 6^x}{6^x} = 6^x $$ $$\left(\frac{7}{6}\right)^x \cdot 7^x – 7^x = 6^x $$
Let $$t = \left(\frac{7}{6}\right)^x $$, then: $$t \cdot 7^x – 7^x = 6^x $$ $$7^x(t – 1) = 6^x $$
Taking the equation $$7^x(t – 1) = 6^x $$: $$7^x(t – 1) = 6^x $$ $$\frac{7^x}{6^x}(t – 1) = 1 $$ $$\left(\frac{7}{6}\right)^x(t – 1) = 1 $$ $$t(t – 1) = 1 $$ $$t^2 – t – 1 = 0 $$
Using the quadratic formula: $$t = \frac{1 \pm \sqrt{1 + 4}}{2} = \frac{1 \pm \sqrt{5}}{2} $$
Since $$t = \left(\frac{7}{6}\right)^x $$ must be positive, both solutions $$t = \frac{1 + \sqrt{5}}{2} $$ and $$t = \frac{1 – \sqrt{5}}{2} $$ are possible if they’re positive.
$$\frac{1 + \sqrt{5}}{2} \approx 1.618 $$ (positive) ✓ $$\frac{1 – \sqrt{5}}{2} \approx -0.618 $$ (negative) ✗
So $$t = \frac{1 + \sqrt{5}}{2} $$.
Now we solve for $$x $$: $$\left(\frac{7}{6}\right)^x = \frac{1 + \sqrt{5}}{2} $$
Taking the natural logarithm of both sides: $$x \ln\left(\frac{7}{6}\right) = \ln\left(\frac{1 + \sqrt{5}}{2}\right) $$ $$x = \frac{\ln\left(\frac{1 + \sqrt{5}}{2}\right)}{\ln\left(\frac{7}{6}\right)} $$
This is the value of $$x $$ that solves the original equation.
SECTION C: TRIGONOMETRY AND COMPLEX NUMBERS
Question 9(a)
Given that $$z_1 = 1 – 2i $$, $$z_2 = 3 + 2i $$ and $$z_3 = 1 + i $$, simplify the following: (i) $$z_1z_2z_3 $$ (ii) $$z_1z_2 – z_2z_3 $$ (iii) $$\frac{z_1z_3}{z_1} $$
Solution:
(i) $$z_1z_2z_3 $$
First, let’s calculate $$z_1z_2 $$: $$z_1z_2 = (1 – 2i)(3 + 2i) $$ $$= 3 + 2i – 6i – 4i^2 $$ $$= 3 – 4i – 4(-1) $$ (since $$i^2 = -1 $$) $$= 3 – 4i + 4 $$ $$= 7 – 4i $$
Now, let’s calculate $$z_1z_2z_3 $$: $$z_1z_2z_3 = (7 – 4i)(1 + i) $$ $$= 7 + 7i – 4i – 4i^2 $$ $$= 7 + 3i – 4(-1) $$ $$= 7 + 3i + 4 $$ $$= 11 + 3i $$
(ii) $$z_1z_2 – z_2z_3 $$
We already calculated $$z_1z_2 = 7 – 4i $$. Now let’s calculate $$z_2z_3 $$: $$z_2z_3 = (3 + 2i)(1 + i) $$ $$= 3 + 3i + 2i + 2i^2 $$ $$= 3 + 5i + 2(-1) $$ $$= 3 + 5i – 2 $$ $$= 1 + 5i $$
Now we calculate $$z_1z_2 – z_2z_3 $$: $$z_1z_2 – z_2z_3 = (7 – 4i) – (1 + 5i) $$ $$= 7 – 4i – 1 – 5i $$ $$= 6 – 9i $$
(iii) $$\frac{z_1z_3}{z_1} $$
Simplifying this expression: $$\frac{z_1z_3}{z_1} = z_3 = 1 + i $$
Question 9(b)
Show that if $$\omega $$ is one complex cube root of unity then $$\omega^2 $$ is also another complex cube root of unity and $$1 + \omega + \omega^2 = 0 $$.
Solution: A complex cube root of unity is a complex number $$z $$ such that $$z^3 = 1 $$.
If $$\omega $$ is a cube root of unity, then $$\omega^3 = 1 $$.
We need to prove that $$\omega^2 $$ is also a cube root of unity: $$(\omega^2)^3 = \omega^6 = (\omega^3)^2 = 1^2 = 1 $$
Therefore, $$\omega^2 $$ is also a cube root of unity.
Now, let’s prove that $$1 + \omega + \omega^2 = 0 $$:
We know that $$\omega^3 = 1 $$, which means $$\omega^3 – 1 = 0 $$. We can factorize $$\omega^3 – 1 $$ as follows: $$\omega^3 – 1 = (\omega – 1)(\omega^2 + \omega + 1) $$
Since $$\omega^3 – 1 = 0 $$ and $$\omega \neq 1 $$ (as $$\omega $$ is a non-trivial cube root of unity), we must have: $$\omega^2 + \omega + 1 = 0 $$
Rearranging: $$1 + \omega + \omega^2 = 0 $$
Which is what we needed to prove.
Question 9(c)
Given that $$z = x + iy $$, show that $$|z + i| = |1 + i| $$ is the circle $$x^2 + y^2 + 2y – 1 = 0 $$.
Solution: We know that $$z = x + iy $$, so $$z + i = x + i(y+1) $$.
The condition $$|z + i| = |1 + i| $$ means: $$|x + i(y+1)| = |1 + i| $$
First, let’s calculate $$|1 + i| $$: $$|1 + i| = \sqrt{1^2 + 1^2} = \sqrt{2} $$
Now, the condition becomes: $$|x + i(y+1)| = \sqrt{2} $$
Squaring both sides: $$|x + i(y+1)|^2 = 2 $$ $$x^2 + (y+1)^2 = 2 $$ $$x^2 + y^2 + 2y + 1 = 2 $$ $$x^2 + y^2 + 2y – 1 = 0 $$
Therefore, the equation of the circle is $$x^2 + y^2 + 2y – 1 = 0 $$.
Question 10(a)
Given that $$\tan A = \frac{1}{7} $$, $$\tan B = \frac{1}{3} $$ and $$A + B + C = 45° $$, find the value of $$\tan C $$.
Solution: We know that $$A + B + C = 45° $$, so $$C = 45° – (A + B) $$.
Using the tangent addition formula: $$\tan(A + B) = \frac{\tan A + \tan B}{1 – \tan A \tan B} $$
$$\tan(A + B) = \frac{\frac{1}{7} + \frac{1}{3}}{1 – \frac{1}{7} \cdot \frac{1}{3}} = \frac{\frac{3 + 7}{21}}{1 – \frac{1}{21}} = \frac{\frac{10}{21}}{\frac{20}{21}} = \frac{10}{20} = \frac{1}{2} $$
Now, using the formula for $$\tan(C) $$ when $$C = 45° – (A + B) $$: $$\tan(45° – (A + B)) = \frac{1 – \tan(A + B)}{1 + \tan(A + B)} $$
$$\tan C = \frac{1 – \frac{1}{2}}{1 + \frac{1}{2}} = \frac{\frac{1}{2}}{\frac{3}{2}} = \frac{1}{3} $$
Therefore, $$\tan C = \frac{1}{3} $$
Question 10(b)
Find the square roots of $$-i $$ and $$3 + 4i $$.
Solution:
- Finding the square roots of $$-i $$:
We need to find a complex number $$z = a + bi $$ such that $$z^2 = -i $$.
Let’s write $$-i $$ in polar form: $$-i = 1e^{-i\pi/2} $$
The square roots are: $$\sqrt{-i} = \sqrt{1}e^{-i\pi/4} = e^{-i\pi/4} = \cos(-\pi/4) + i\sin(-\pi/4) = \frac{\sqrt{2}}{2} – i\frac{\sqrt{2}}{2} $$
and $$\sqrt{-i} = \sqrt{1}e^{-i\pi/4 + i\pi} = e^{i(3\pi/4)} = \cos(3\pi/4) + i\sin(3\pi/4) = -\frac{\sqrt{2}}{2} + i\frac{\sqrt{2}}{2} $$
Therefore, the square roots of $$-i $$ are $$\frac{\sqrt{2}}{2} – i\frac{\sqrt{2}}{2} $$ and $$-\frac{\sqrt{2}}{2} + i\frac{\sqrt{2}}{2} $$
PAPER 2 SECTION A (20%)
1. Evaluate
∫π4π22x(tan2x−sec2x)dx\int_{\frac{\pi}{4}}^{\frac{\pi}{2}} 2x \left( \tan^2 x – \sec^2 x \right) dx
Solution:
Recall:
tan2x=sec2x−1⇒tan2x−sec2x=−1\tan^2 x = \sec^2 x – 1 \Rightarrow \tan^2 x – \sec^2 x = -1
So the integral becomes:
∫π4π22x(−1) dx=−2∫π4π2x dx=−2[x22]π4π2=−[x2]π4π2\int_{\frac{\pi}{4}}^{\frac{\pi}{2}} 2x(-1)\,dx = -2 \int_{\frac{\pi}{4}}^{\frac{\pi}{2}} x\,dx = -2 \left[ \frac{x^2}{2} \right]_{\frac{\pi}{4}}^{\frac{\pi}{2}} = -\left[ x^2 \right]_{\frac{\pi}{4}}^{\frac{\pi}{2}} =−((π2)2−(π4)2)=−(π24−π216)=−(4π2−π216)=−3π216= -\left( \left( \frac{\pi}{2} \right)^2 – \left( \frac{\pi}{4} \right)^2 \right) = -\left( \frac{\pi^2}{4} – \frac{\pi^2}{16} \right) = -\left( \frac{4\pi^2 – \pi^2}{16} \right) = -\frac{3\pi^2}{16}
2. Find the sine of the angle between the vectors
a⃗=3i−2j+k,b⃗=i+2j−3k\vec{a} = 3\mathbf{i} – 2\mathbf{j} + \mathbf{k}, \quad \vec{b} = \mathbf{i} + 2\mathbf{j} – 3\mathbf{k}
Solution:
Use the formula:
sinθ=∣a⃗×b⃗∣∣a⃗∣∣b⃗∣\sin \theta = \frac{|\vec{a} \times \vec{b}|}{|\vec{a}||\vec{b}|}
Compute a⃗×b⃗\vec{a} \times \vec{b}:
a⃗×b⃗=∣ijk3−2112−3∣=i((−2)(−3)−(1)(2))−j(3(−3)−(1)(1))+k(3(2)−(−2)(1))\vec{a} \times \vec{b} = \begin{vmatrix} \mathbf{i} & \mathbf{j} & \mathbf{k} \\ 3 & -2 & 1 \\ 1 & 2 & -3 \\ \end{vmatrix} = \mathbf{i}((-2)(-3) – (1)(2)) – \mathbf{j}(3(-3) – (1)(1)) + \mathbf{k}(3(2) – (-2)(1)) =i(6−2)−j(−9−1)+k(6+2)=4i+10j+8k= \mathbf{i}(6 – 2) – \mathbf{j}(-9 – 1) + \mathbf{k}(6 + 2) = 4\mathbf{i} + 10\mathbf{j} + 8\mathbf{k} ∣a⃗×b⃗∣=42+102+82=16+100+64=180|\vec{a} \times \vec{b}| = \sqrt{4^2 + 10^2 + 8^2} = \sqrt{16 + 100 + 64} = \sqrt{180} ∣a⃗∣=32+(−2)2+12=9+4+1=14|\vec{a}| = \sqrt{3^2 + (-2)^2 + 1^2} = \sqrt{9 + 4 + 1} = \sqrt{14} ∣b⃗∣=12+22+(−3)2=1+4+9=14|\vec{b}| = \sqrt{1^2 + 2^2 + (-3)^2} = \sqrt{1 + 4 + 9} = \sqrt{14} sinθ=18014⋅14=18014=6514=357\sin \theta = \frac{\sqrt{180}}{\sqrt{14} \cdot \sqrt{14}} = \frac{\sqrt{180}}{14} = \frac{6\sqrt{5}}{14} = \frac{3\sqrt{5}}{7}
3. Solve the equation
dydx=e2x+y\frac{dy}{dx} = e^{2x + y}
Solution:
Separate variables:
e−ydy=e2xdx⇒∫e−ydy=∫e2xdxe^{-y} dy = e^{2x} dx \Rightarrow \int e^{-y} dy = \int e^{2x} dx −e−y=12e2x+C⇒e−y=−12e2x−C⇒y=−ln(−12e2x−C)- e^{-y} = \frac{1}{2} e^{2x} + C \Rightarrow e^{-y} = -\frac{1}{2} e^{2x} – C \Rightarrow y = -\ln \left( -\frac{1}{2} e^{2x} – C \right)
4. Differentiate esinxcos2xe^{\sin x} \cos 2x
Solution:
Let y=esinxcos2xy = e^{\sin x} \cos 2x
Use product rule:
dydx=esinx⋅cos2x⋅cosx−2esinx⋅sin2x=esinx(cosx⋅cos2x−2sin2x)\frac{dy}{dx} = e^{\sin x} \cdot \cos 2x \cdot \cos x – 2e^{\sin x} \cdot \sin 2x = e^{\sin x} \left( \cos x \cdot \cos 2x – 2 \sin 2x \right)
5. Find the value of xx for which the vectors
xi−28j+k,xi−25kx\mathbf{i} – 28\mathbf{j} + \mathbf{k}, \quad x\mathbf{i} – 25\mathbf{k}
are perpendicular.
Solution:
Vectors are perpendicular if dot product = 0.
Let:
u⃗=xi−28j+k,v⃗=xi−25k\vec{u} = x\mathbf{i} – 28\mathbf{j} + \mathbf{k}, \quad \vec{v} = x\mathbf{i} – 25\mathbf{k} u⃗⋅v⃗=x⋅x+0⋅(−28)+1⋅(−25)=x2−25\vec{u} \cdot \vec{v} = x \cdot x + 0 \cdot (-28) + 1 \cdot (-25) = x^2 – 25
Set equal to 0:
x2−25=0⇒x=±5x^2 – 25 = 0 \Rightarrow x = \pm 5
Let’s continue with SECTION B: CALCULUS of the IJMB 2024 A-Level Mathematics Paper II.
SECTION B (40%)
6(a)
If
f(x)={x2+3x−2,x<−1×3+3×2−x+1,x≥−1f(x) = \begin{cases} x^2 + 3x – 2, & x < -1 \\ x^3 + 3x^2 – x + 1, & x \geq -1 \end{cases}
Determine whether f(x)f(x) is continuous at x=−1x = -1.
Solution:
A function is continuous at x=−1x = -1 if:
limx→−1−f(x)=limx→−1+f(x)=f(−1)\lim_{x \to -1^-} f(x) = \lim_{x \to -1^+} f(x) = f(-1)
Left-hand limit:
Use the first part (since x<−1x < -1):
limx→−1−f(x)=(−1)2+3(−1)−2=1−3−2=−4\lim_{x \to -1^-} f(x) = (-1)^2 + 3(-1) – 2 = 1 – 3 – 2 = -4
Right-hand limit and f(−1)f(-1):
Use second part (since x≥−1x \geq -1):
limx→−1+f(x)=(−1)3+3(−1)2−(−1)+1=−1+3+1+1=4\lim_{x \to -1^+} f(x) = (-1)^3 + 3(-1)^2 – (-1) + 1 = -1 + 3 + 1 + 1 = 4
Since −4≠4-4 \neq 4,
limx→−1−f(x)≠limx→−1+f(x)\lim_{x \to -1^-} f(x) \neq \lim_{x \to -1^+} f(x)
Therefore, f(x)f(x) is not continuous at x=−1x = -1.
6(b)
Find the coordinates of the turning point(s) of
y=2×3−15×2+36x−1y = 2x^3 – 15x^2 + 36x – 1
Solution:
Find dydx\frac{dy}{dx}:
dydx=6×2−30x+36\frac{dy}{dx} = 6x^2 – 30x + 36
Set derivative to 0:
6×2−30x+36=0⇒x2−5x+6=0⇒(x−2)(x−3)=0⇒x=2, 36x^2 – 30x + 36 = 0 \Rightarrow x^2 – 5x + 6 = 0 \Rightarrow (x – 2)(x – 3) = 0 \Rightarrow x = 2, \, 3
Find y-values:
For x=2x = 2:
y=2(8)−15(4)+36(2)−1=16−60+72−1=27y = 2(8) – 15(4) + 36(2) – 1 = 16 – 60 + 72 – 1 = 27
For x=3x = 3:
y=2(27)−15(9)+36(3)−1=54−135+108−1=26y = 2(27) – 15(9) + 36(3) – 1 = 54 – 135 + 108 – 1 = 26
Coordinates of turning points:
- (2,27)(2, 27)
- (3,26)(3, 26)
Determine nature of turning points:
Second derivative:
d2ydx2=12x−30\frac{d^2y}{dx^2} = 12x – 30
At x=2x = 2: 12(2)−30=−612(2) – 30 = -6 → Maximum
At x=3x = 3: 12(3)−30=612(3) – 30 = 6 → Minimum
7(a)
Find the volume generated when the region bounded by y=4x−x2y = 4x – x^2, y=0y = 0, and the y-axis is rotated about the x-axis.
Solution:
The region is from x=0x = 0 to where y=0y = 0:
4x−x2=0⇒x(4−x)=0⇒x=0, 44x – x^2 = 0 \Rightarrow x(4 – x) = 0 \Rightarrow x = 0, \, 4
Volume of revolution about x-axis:
V=π∫04(4x−x2)2dxV = \pi \int_0^4 (4x – x^2)^2 dx
First expand:
(4x−x2)2=16×2−8×3+x4(4x – x^2)^2 = 16x^2 – 8x^3 + x^4 V=π∫04(x4−8×3+16×2)dx=π[x55−2×4+16×33]04V = \pi \int_0^4 (x^4 – 8x^3 + 16x^2) dx = \pi \left[ \frac{x^5}{5} – 2x^4 + \frac{16x^3}{3} \right]_0^4
At x=4x = 4:
455=10245,2(44)=2(256)=512,16(64)3=10243\frac{4^5}{5} = \frac{1024}{5}, \quad 2(4^4) = 2(256) = 512, \quad \frac{16(64)}{3} = \frac{1024}{3} V=π(10245−512+10243)V = \pi \left( \frac{1024}{5} – 512 + \frac{1024}{3} \right)
Find LCM of 5, 1, 3 = 15:
=π(307215−768015+512015)=π⋅51215= \pi \left( \frac{3072}{15} – \frac{7680}{15} + \frac{5120}{15} \right) = \pi \cdot \frac{512}{15}
So,
V=512π15 units3V = \frac{512\pi}{15} \text{ units}^3
7(b)
Find the equation of the tangent to
y=ln(x2+3x−2)at x=1y = \ln (x^2 + 3x – 2) \quad \text{at } x = 1
Solution:
Let’s find the derivative:
dydx=1×2+3x−2⋅(2x+3)\frac{dy}{dx} = \frac{1}{x^2 + 3x – 2} \cdot (2x + 3)
At x=1x = 1:
x2+3x−2=1+3−2=2,2x+3=5⇒dydx=52x^2 + 3x – 2 = 1 + 3 – 2 = 2,\quad 2x + 3 = 5 \Rightarrow \frac{dy}{dx} = \frac{5}{2}
Point on the curve:
y=ln(2)⇒Point is (1,ln2)y = \ln(2) \Rightarrow \text{Point is } (1, \ln 2)
Tangent:
y−ln2=52(x−1)⇒y=52x−52+ln2y – \ln 2 = \frac{5}{2}(x – 1) \Rightarrow y = \frac{5}{2}x – \frac{5}{2} + \ln 2
SECTION C (20%)
8(a)
Solve the differential equation:
dydx=y+xy−x\frac{dy}{dx} = \frac{y + x}{y – x}
Solution:
Let’s use the substitution method.
Let
v=yx⇒y=vx⇒dydx=v+xdvdxv = \frac{y}{x} \Rightarrow y = vx \Rightarrow \frac{dy}{dx} = v + x\frac{dv}{dx}
Substitute into the equation:
v+xdvdx=vx+xvx−x=x(v+1)x(v−1)=v+1v−1v + x\frac{dv}{dx} = \frac{vx + x}{vx – x} = \frac{x(v + 1)}{x(v – 1)} = \frac{v + 1}{v – 1}
Subtract vv from both sides:
xdvdx=v+1v−1−v=v+1−v(v−1)v−1=v+1−(v2−v)v−1=v+1−v2+vv−1=−v2+2v+1v−1x\frac{dv}{dx} = \frac{v + 1}{v – 1} – v = \frac{v + 1 – v(v – 1)}{v – 1} = \frac{v + 1 – (v^2 – v)}{v – 1} = \frac{v + 1 – v^2 + v}{v – 1} = \frac{-v^2 + 2v + 1}{v – 1}
So,
xdvdx=−v2+2v+1v−1x\frac{dv}{dx} = \frac{-v^2 + 2v + 1}{v – 1}
Let’s integrate:
v−1−v2+2v+1dv=1xdx\frac{v – 1}{-v^2 + 2v + 1} dv = \frac{1}{x} dx
Factor denominator:
−v2+2v+1=−(v2−2v−1)-v^2 + 2v + 1 = -(v^2 – 2v – 1)
This integral is a bit messy, so alternatively, use substitution earlier:
Try:
dydx=y+xy−x⇒Homogeneous equation, try v=yx\frac{dy}{dx} = \frac{y + x}{y – x} \Rightarrow \text{Homogeneous equation, try } v = \frac{y}{x}
So y=vx⇒dydx=v+xdvdxy = vx \Rightarrow \frac{dy}{dx} = v + x\frac{dv}{dx}
Substitute into original:
v+xdvdx=vx+xvx−x=x(v+1)x(v−1)=v+1v−1⇒xdvdx=v+1v−1−v=−v2+2v+1v−1⇒v−1−v2+2v+1dv=1xdxv + x\frac{dv}{dx} = \frac{vx + x}{vx – x} = \frac{x(v + 1)}{x(v – 1)} = \frac{v + 1}{v – 1} \Rightarrow x\frac{dv}{dx} = \frac{v + 1}{v – 1} – v = \frac{-v^2 + 2v + 1}{v – 1} \Rightarrow \frac{v – 1}{-v^2 + 2v + 1} dv = \frac{1}{x} dx
Now integrate both sides (LHS is rational, partial fractions can be used):
8(b)
Find the magnitude and direction of the vector
v⃗=3i−4j\vec{v} = 3\mathbf{i} – 4\mathbf{j}
Solution:
Magnitude:
∣v⃗∣=32+(−4)2=9+16=25=5|\vec{v}| = \sqrt{3^2 + (-4)^2} = \sqrt{9 + 16} = \sqrt{25} = 5
Direction (angle with positive x-axis):
θ=tan−1(−43)=−53.13∘\theta = \tan^{-1} \left( \frac{-4}{3} \right) = -53.13^\circ
So the direction is 53.13° below the x-axis (or 306.87∘306.87^\circ from the positive x-axis in standard position).
9(a)
If the acceleration of a particle is given by
d2xdt2=6t, with initial velocity 5 and initial displacement 2,\frac{d^2x}{dt^2} = 6t, \text{ with initial velocity } 5 \text{ and initial displacement } 2,
find the displacement as a function of time.
Solution:
Integrate acceleration to get velocity:
dvdt=6t⇒v=∫6t dt=3t2+C\frac{dv}{dt} = 6t \Rightarrow v = \int 6t \, dt = 3t^2 + C
Given: initial velocity v(0)=5v(0) = 5
v=3t2+C⇒5=0+C⇒C=5⇒v=3t2+5v = 3t^2 + C \Rightarrow 5 = 0 + C \Rightarrow C = 5 \Rightarrow v = 3t^2 + 5
Now integrate velocity to get displacement:
x=∫(3t2+5)dt=t3+5t+C′x = \int (3t^2 + 5) dt = t^3 + 5t + C’
Given: x(0)=2⇒2=0+0+C′⇒C′=2x(0) = 2 \Rightarrow 2 = 0 + 0 + C’ \Rightarrow C’ = 2
Final answer:
x(t)=t3+5t+2x(t) = t^3 + 5t + 2
9(b)
Two forces: F⃗1=2i+3j\vec{F}_1 = 2\mathbf{i} + 3\mathbf{j},
F⃗2=−i+4j\vec{F}_2 = -\mathbf{i} + 4\mathbf{j}.
Find the resultant force and its magnitude.
Solution:
Resultant:
R⃗=F⃗1+F⃗2=(2−1)i+(3+4)j=i+7j\vec{R} = \vec{F}_1 + \vec{F}_2 = (2 – 1)\mathbf{i} + (3 + 4)\mathbf{j} = \mathbf{i} + 7\mathbf{j}
Magnitude:
∣R⃗∣=12+72=1+49=50=52|\vec{R}| = \sqrt{1^2 + 7^2} = \sqrt{1 + 49} = \sqrt{50} = 5\sqrt{2}
Direction:
θ=tan−1(71)=tan−1(7)≈81.87∘\theta = \tan^{-1}\left( \frac{7}{1} \right) = \tan^{-1}(7) \approx 81.87^\circ
Continuing with the last question of the IJMB 2024 A-Level Mathematics Paper II – SECTION C:
10(a)
Solve the differential equation:
dydx=y2sinx\frac{dy}{dx} = y^2 \sin x
Solution:
This is a separable differential equation.
dydx=y2sinx⇒1y2dy=sinx dx\frac{dy}{dx} = y^2 \sin x \Rightarrow \frac{1}{y^2} dy = \sin x \, dx
Integrate both sides:
∫y−2dy=∫sinx dx⇒−1y=−cosx+C\int y^{-2} dy = \int \sin x \, dx \Rightarrow -\frac{1}{y} = -\cos x + C
Multiply both sides by –1:
1y=cosx−C⇒y=1cosx−C\frac{1}{y} = \cos x – C \Rightarrow y = \frac{1}{\cos x – C}
Final Answer:
y(x)=1cosx−Cy(x) = \frac{1}{\cos x – C}
10(b)
Given vectors a⃗=3i+2j\vec{a} = 3\mathbf{i} + 2\mathbf{j}, b⃗=i+4j\vec{b} = \mathbf{i} + 4\mathbf{j},
find a unit vector in the direction of a⃗+b⃗\vec{a} + \vec{b}.
Solution:
First, find a⃗+b⃗\vec{a} + \vec{b}:
a⃗+b⃗=(3+1)i+(2+4)j=4i+6j\vec{a} + \vec{b} = (3 + 1)\mathbf{i} + (2 + 4)\mathbf{j} = 4\mathbf{i} + 6\mathbf{j}
Magnitude of a⃗+b⃗\vec{a} + \vec{b}:
∣a⃗+b⃗∣=42+62=16+36=52=213|\vec{a} + \vec{b}| = \sqrt{4^2 + 6^2} = \sqrt{16 + 36} = \sqrt{52} = 2\sqrt{13}
Now divide by the magnitude to get the unit vector:
u^=1213(4i+6j)=4213i+6213j=213i+313j\hat{u} = \frac{1}{2\sqrt{13}}(4\mathbf{i} + 6\mathbf{j}) = \frac{4}{2\sqrt{13}}\mathbf{i} + \frac{6}{2\sqrt{13}}\mathbf{j} = \frac{2}{\sqrt{13}}\mathbf{i} + \frac{3}{\sqrt{13}}\mathbf{j}
Final Answer:
u^=213i+313j\boxed{\hat{u} = \frac{2}{\sqrt{13}}\mathbf{i} + \frac{3}{\sqrt{13}}\mathbf{j}}
SECTION A
Question 1: Calculate the gradient of the line joining the points (7, 50) and (-7, 8).
To find the gradient of a line joining two points $$(x_1, y_1) $$ and $$(x_2, y_2) $$, we use the formula:
$$ $$m = \frac{y_2 – y_1}{x_2 – x_1} $$ $$
Given:
– Point 1: $$(7, 50) $$
– Point 2: $$(-7, 8) $$
Substituting these values:
$$ $$m = \frac{8 – 50}{-7 – 7} = \frac{-42}{-14} = \frac{42}{14} = 3 $$ $$
Therefore, the gradient of the line joining the points $$(7, 50) $$ and $$(-7, 8) $$ is $$3 $$.
Question 2: Calculate the mean of the distribution f(x)=e−3xf(x) = e^{-3x} f(x)=e−3x where 0≤x<∞0 \leq x < \infty 0≤x<∞.
For a continuous probability distribution $$f(x) $$, the mean (expected value) is calculated using:
$$ $$\mu = \int_{-\infty}^{\infty} x \cdot f(x) \, dx $$ $$
For our distribution $$f(x) = e^{-3x} $$ where $$0 \leq x < \infty $$, we first need to verify if it’s a valid probability density function by checking if $$\int_{0}^{\infty} e^{-3x} \, dx = 1 $$.
$$ $$\int_{0}^{\infty} e^{-3x} \, dx = \left[ -\frac{1}{3} e^{-3x} \right]_{0}^{\infty} = -\frac{1}{3} \left( 0 – 1 \right) = \frac{1}{3} $$ $$
Since this doesn’t equal 1, we need to normalize by multiplying by 3:
$$ $$f(x) = 3e^{-3x} $$ $$
Now, we can calculate the mean:
$$ $$\mu = \int_{0}^{\infty} x \cdot 3e^{-3x} \, dx = 3 \int_{0}^{\infty} x \cdot e^{-3x} \, dx $$ $$
Using integration by parts with $$u = x $$ and $$dv = e^{-3x} \, dx $$:
$$ $$\mu = 3 \left[ -\frac{x}{3} e^{-3x} \right]_{0}^{\infty} + 3 \int_{0}^{\infty} \frac{1}{3} e^{-3x} \, dx $$ $$
$$ $$\mu = 3 \left( 0 + 0 \right) + 3 \cdot \frac{1}{3} \cdot \frac{1}{3} = \frac{1}{3} $$ $$
Therefore, the mean of the distribution is $$\frac{1}{3} $$.
Question 3: Find the equation of the line through the mid-point of (2,3), (-5,1) and perpendicular to the line 2x+y=7002x + y = 700 2x+y=700.
Step 1: Find the mid-point of $$(2,3) $$ and $$(-5,1) $$.
The mid-point formula is: $$(\frac{x_1 + x_2}{2}, \frac{y_1 + y_2}{2}) $$
Mid-point $$= (\frac{2 + (-5)}{2}, \frac{3 + 1}{2}) = (\frac{-3}{2}, \frac{4}{2}) = (-\frac{3}{2}, 2) $$
Step 2: Find the slope of the line $$2x + y = 700 $$.
Rewriting in slope-intercept form: $$y = -2x + 700 $$
So, the slope of this line is $$-2 $$.
Step 3: The slope of a perpendicular line is the negative reciprocal of the original slope.
Since the slope of $$2x + y = 700 $$ is $$-2 $$, the slope of the perpendicular line is $$\frac{1}{2} $$.
Step 4: Use the point-slope form of a line equation.
The point-slope form is: $$y – y_1 = m(x – x_1) $$
where $$(x_1, y_1) $$ is the point $$(-\frac{3}{2}, 2) $$ and $$m = \frac{1}{2} $$
$$ $$y – 2 = \frac{1}{2}(x – (-\frac{3}{2})) $$ $$
$$ $$y – 2 = \frac{1}{2}(x + \frac{3}{2}) $$ $$
$$ $$y – 2 = \frac{1}{2}x + \frac{3}{4} $$ $$
$$ $$y = \frac{1}{2}x + \frac{3}{4} + 2 $$ $$
$$ $$y = \frac{1}{2}x + \frac{3}{4} + \frac{8}{4} $$ $$
$$ $$y = \frac{1}{2}x + \frac{11}{4} $$ $$
Therefore, the equation of the required line is $$y = \frac{1}{2}x + \frac{11}{4} $$ or $$2y = x + \frac{11}{2} $$ or $$2y – x = \frac{11}{2} $$.
Question 4: Find the equation of the circle whose end points of diameter are (7, -3) and (-1,2).
To find the equation of a circle given the endpoints of a diameter:
Step 1: Find the center of the circle, which is the midpoint of the diameter.
Center $$= (\frac{7 + (-1)}{2}, \frac{-3 + 2}{2}) = (\frac{6}{2}, \frac{-1}{2}) = (3, -\frac{1}{2}) $$
Step 2: Calculate the radius, which is half the distance between the endpoints.
$$ $$r = \frac{1}{2} \sqrt{(7-(-1))^2 + (-3-2)^2} = \frac{1}{2} \sqrt{8^2 + (-5)^2} = \frac{1}{2} \sqrt{64 + 25} = \frac{1}{2} \sqrt{89} $$ $$
Step 3: Write the standard equation of a circle.
The standard form is: $$(x – h)^2 + (y – k)^2 = r^2 $$, where $$(h, k) $$ is the center.
$$ $$(x – 3)^2 + (y – (-\frac{1}{2}))^2 = (\frac{\sqrt{89}}{2})^2 $$ $$
$$ $$(x – 3)^2 + (y + \frac{1}{2})^2 = \frac{89}{4} $$ $$
Therefore, the equation of the circle is $$(x – 3)^2 + (y + \frac{1}{2})^2 = \frac{89}{4} $$.
Question 5: Find the coordinates of the vertex and focus of the parabola x2=16−4yx^2 = 16 – 4y x2=16−4y.
Let’s rewrite the equation to match the standard form for a parabola.
$$ $$x^2 = 16 – 4y $$ $$
$$ $$x^2 = -4(y – 4) $$ $$
This is in the form $$x^2 = -4a(y – k) $$ where the axis of symmetry is parallel to the y-axis.
Comparing with the standard form, we have $$a = 1 $$ and $$k = 4 $$.
For a parabola with equation $$x^2 = -4a(y – k) $$:
– The vertex is at $$(0, k) $$
– The focus is at $$(0, k-a) $$
– The directrix is the line $$y = k+a $$
Therefore:
– The vertex is at $$(0, 4) $$
– The focus is at $$(0, 4-1) = (0, 3) $$
– The directrix is the line $$y = 4+1 = 5 $$
I’ll continue with more questions if you’d like. Would you prefer I solve more questions from Section A, or should I proceed to questions from Sections B and C?
Retry
B
continue
Edit
I’ll continue with the solutions for the remaining questions, moving on to Section B.
SECTION B: COORDINATE GEOMETRY
Question 6(a): Obtain the equation of the conic whose parametric equations are given by x=1+2tanθx = 1 + 2 \tan \theta x=1+2tanθ and y=−1+3secθy = -1 + 3 \sec \theta y=−1+3secθ, and describe it completely.
We need to eliminate the parameter $$\theta $$ to find the Cartesian equation of the curve.
Step 1: From the first equation, solve for $$\tan \theta $$.
$$ $$x = 1 + 2\tan \theta $$ $$
$$ $$\tan \theta = \frac{x-1}{2} $$ $$
Step 2: Use the identity $$\sec^2 \theta = 1 + \tan^2 \theta $$
$$ $$\sec^2 \theta = 1 + \left(\frac{x-1}{2}\right)^2 = 1 + \frac{(x-1)^2}{4} = \frac{4 + (x-1)^2}{4} $$ $$
Step 3: From the second equation, solve for $$\sec \theta $$.
$$ $$y = -1 + 3\sec \theta $$ $$
$$ $$\sec \theta = \frac{y+1}{3} $$ $$
Step 4: Square both sides.
$$ $$\sec^2 \theta = \left(\frac{y+1}{3}\right)^2 = \frac{(y+1)^2}{9} $$ $$
Step 5: Equate the expressions for $$\sec^2 \theta $$ from Steps 2 and 4.
$$ $$\frac{4 + (x-1)^2}{4} = \frac{(y+1)^2}{9} $$ $$
Step 6: Cross multiply.
$$ $$9(4 + (x-1)^2) = 4(y+1)^2 $$ $$
$$ $$36 + 9(x-1)^2 = 4(y+1)^2 $$ $$
$$ $$36 + 9(x^2 – 2x + 1) = 4(y^2 + 2y + 1) $$ $$
$$ $$36 + 9x^2 – 18x + 9 = 4y^2 + 8y + 4 $$ $$
$$ $$9x^2 – 18x – 4y^2 – 8y + 41 = 0 $$ $$
This is the equation of a hyperbola. To analyze it completely:
Let’s convert it to standard form by completing the squares:
$$ $$9(x^2 – 2x + 1) – 4(y^2 + 2y + 1) = -41 + 9 – 4 $$ $$
$$ $$9(x – 1)^2 – 4(y + 1)^2 = -36 $$ $$
$$ $$\frac{9(x – 1)^2}{36} – \frac{4(y + 1)^2}{36} = -1 $$ $$
$$ $$\frac{(x – 1)^2}{4} – \frac{(y + 1)^2}{9} = -1 $$ $$
Taking the negative of both sides:
$$ $$\frac{(y + 1)^2}{9} – \frac{(x – 1)^2}{4} = 1 $$ $$
This is a hyperbola with:
– Center at $$(1, -1) $$
– Transverse axis parallel to the y-axis
– Semi-major axis $$a = 3 $$
– Semi-minor axis $$b = 2 $$
– Eccentricity $$e = \sqrt{1 + \frac{b^2}{a^2}} = \sqrt{1 + \frac{4}{9}} = \sqrt{\frac{13}{9}} = \frac{\sqrt{13}}{3} $$
– Foci at $$(1, -1 \pm c) $$ where $$c = ae = 3 \cdot \frac{\sqrt{13}}{3} = \sqrt{13} $$
Question 6(b): Sketch the curve r=(1+sinθ)r = (1 + \sin \theta) r=(1+sinθ) in polar form for 0≤θ≤360°0 \leq \theta \leq 360° 0≤θ≤360°.
The equation $$r = 1 + \sin \theta $$ represents a cardioid (heart-shaped curve).
Key points to plot:
– At $$\theta = 0° $$: $$r = 1 + \sin(0) = 1 + 0 = 1 $$
– At $$\theta = 90° $$: $$r = 1 + \sin(90°) = 1 + 1 = 2 $$ (maximum distance)
– At $$\theta = 180° $$: $$r = 1 + \sin(180°) = 1 + 0 = 1 $$
– At $$\theta = 270° $$: $$r = 1 + \sin(270°) = 1 + (-1) = 0 $$ (curve passes through origin)
– At $$\theta = 360° $$: $$r = 1 + \sin(360°) = 1 + 0 = 1 $$
The curve looks like a heart shape with a cusp at the origin. It has:
– Maximum distance from origin: 2 units at $$\theta = 90° $$
– Minimum distance from origin: 0 units at $$\theta = 270° $$
– The curve passes through the point $$(1,0) $$ when $$\theta = 0° $$ or $$\theta = 180° $$
Question 6(c): Show that r=11+cosθr = \frac{1}{1+\cos \theta} r=1+cosθ1 is the polar form of a parabola.
To show that $$r = \frac{1}{1+\cos \theta} $$ represents a parabola, we need to convert it to Cartesian form and verify it matches the standard form of a parabola.
Step 1: Recall the relationships between polar and Cartesian coordinates:
– $$x = r\cos \theta $$
– $$y = r\sin \theta $$
– $$r^2 = x^2 + y^2 $$
Step 2: From the given equation $$r = \frac{1}{1+\cos \theta} $$, we have:
$$ $$r(1+\cos \theta) = 1 $$ $$
$$ $$r + r\cos \theta = 1 $$ $$
Step 3: Substitute $$r\cos \theta = x $$:
$$ $$r + x = 1 $$ $$
$$ $$r = 1 – x $$ $$
Step 4: Now use $$r^2 = x^2 + y^2 $$:
$$ $$(1-x)^2 = x^2 + y^2 $$ $$
$$ $$1 – 2x + x^2 = x^2 + y^2 $$ $$
$$ $$1 – 2x = y^2 $$ $$
$$ $$y^2 = -2x + 1 $$ $$
This is the standard form of a parabola with:
– Vertex at $$(\frac{1}{2}, 0) $$
– Focus at $$(0,0) $$ (the origin in the polar coordinate system)
– Directrix is the vertical line $$x = 1 $$
– The parabola opens to the left (negative x-direction)
Therefore, $$r = \frac{1}{1+\cos \theta} $$ is indeed the polar form of a parabola.
Question 7(a): Show that the equation 9×2+25y2−54x−100y−44=09x^2 + 25y^2 – 54x – 100y – 44 = 0 9×2+25y2−54x−100y−44=0 represents an Ellipse. Find the coordinates of its center.
First, let’s rearrange to group the terms with $$x $$ and $$y $$:
$$ $$9x^2 – 54x + 25y^2 – 100y – 44 = 0 $$ $$
Now complete the squares for both $$x $$ and $$y $$ terms:
For the $$x $$ terms:
$$ $$9x^2 – 54x = 9(x^2 – 6x) = 9(x^2 – 6x + 9 – 9) = 9(x – 3)^2 – 81 $$ $$
For the $$y $$ terms:
$$ $$25y^2 – 100y = 25(y^2 – 4y) = 25(y^2 – 4y + 4 – 4) = 25(y – 2)^2 – 100 $$ $$
Substituting back:
$$ $$9(x – 3)^2 – 81 + 25(y – 2)^2 – 100 – 44 = 0 $$ $$
$$ $$9(x – 3)^2 + 25(y – 2)^2 = 81 + 100 + 44 = 225 $$ $$
$$ $$\frac{9(x – 3)^2}{225} + \frac{25(y – 2)^2}{225} = 1 $$ $$
$$ $$\frac{(x – 3)^2}{25} + \frac{(y – 2)^2}{9} = 1 $$ $$
This is the standard form of an ellipse $$\frac{(x-h)^2}{a^2} + \frac{(y-k)^2}{b^2} = 1 $$ with:
– Center at $$(h,k) = (3,2) $$
– Semi-major axis $$a = 5 $$ (along the x-axis)
– Semi-minor axis $$b = 3 $$ (along the y-axis)
Therefore, the equation represents an ellipse with center at $$(3,2) $$.
Question 7(b): Find the equation of a straight line parallel to the line x−3y=5x – 3y = 5 x−3y=5 which passes through the mid-point of the line joining the points (3,−2)(3,-2) (3,−2) and (7,8)(7,8) (7,8).
Step 1: Find the mid-point of the line joining $$(3,-2) $$ and $$(7,8) $$.
Mid-point $$= \left(\frac{3+7}{2}, \frac{-2+8}{2}\right) = \left(\frac{10}{2}, \frac{6}{2}\right) = (5, 3) $$
Step 2: The slope of the line $$x – 3y = 5 $$ is determined by rearranging to slope-intercept form:
$$ $$x – 3y = 5 $$ $$
$$ $$-3y = -x + 5 $$ $$
$$ $$y = \frac{1}{3}x – \frac{5}{3} $$ $$
So the slope of the given line is $$\frac{1}{3} $$.
Step 3: Since we want a line parallel to $$x – 3y = 5 $$, our new line will have the same slope: $$\frac{1}{3} $$.
Step 4: Use the point-slope form to find the equation of the new line:
$$ $$y – y_1 = m(x – x_1) $$ $$
$$ $$y – 3 = \frac{1}{3}(x – 5) $$ $$
$$ $$y – 3 = \frac{1}{3}x – \frac{5}{3} $$ $$
$$ $$y = \frac{1}{3}x – \frac{5}{3} + 3 $$ $$
$$ $$y = \frac{1}{3}x + \frac{4}{3} $$ $$
Converting back to general form:
$$ $$3y = x + 4 $$ $$
$$ $$x – 3y = -4 $$ $$
Therefore, the equation of the required line is $$x – 3y = -4 $$.
Question 7(c): Find the value of 2x+y2x + y 2x+y, if (2,3)(2,3) (2,3) is the mid-point of (x,3)(x,3) (x,3) and (−3,y)(-3,y) (−3,y).
If $$(2,3) $$ is the mid-point of $$(x,3) $$ and $$(-3,y) $$, then:
$$ $$2 = \frac{x + (-3)}{2} $$ $$
$$ $$3 = \frac{3 + y}{2} $$ $$
Solving for $$x $$ from the first equation:
$$ $$2 = \frac{x – 3}{2} $$ $$
$$ $$4 = x – 3 $$ $$
$$ $$x = 7 $$ $$
Solving for $$y $$ from the second equation:
$$ $$3 = \frac{3 + y}{2} $$ $$
$$ $$6 = 3 + y $$ $$
$$ $$y = 3 $$ $$
Now we can find the value of $$2x + y $$:
$$ $$2x + y = 2(7) + 3 = 14 + 3 = 17 $$ $$
Therefore, the value of $$2x + y $$ is $$17 $$.
Question 8(a): Determine the condition for the line y=mxy = mx y=mx to be tangent to the parabola y2=4a(x−5)y^2 = 4a(x – 5) y2=4a(x−5). Write down the general equation of the tangent to the parabola.
A line is tangent to a parabola if it intersects the parabola at exactly one point.
Step 1: Find the intersection of the line $$y = mx $$ and the parabola $$y^2 = 4a(x – 5) $$.
Substitute $$y = mx $$ into the parabola equation:
$$ $$m^2x^2 = 4a(x – 5) $$ $$
$$ $$m^2x^2 = 4ax – 20a $$ $$
$$ $$m^2x^2 – 4ax + 20a = 0 $$ $$
Step 2: For the line to be tangent, this quadratic equation must have exactly one solution (a repeated root). This occurs when the discriminant is zero:
$$ $$b^2 – 4ac = 0 $$ $$
$$ $$(-4a)^2 – 4(m^2)(20a) = 0 $$ $$
$$ $$16a^2 – 80am^2 = 0 $$ $$
$$ $$16a^2 = 80am^2 $$ $$
$$ $$a = 5m^2 $$ $$
Therefore, the condition for the line $$y = mx $$ to be tangent to the parabola is $$a = 5m^2 $$.
Step 3: For the general equation of a tangent to the parabola $$y^2 = 4a(x – 5) $$, we can use the formula:
For a point $$(x_0, y_0) $$ on the parabola, the equation of the tangent at that point is:
$$ $$y y_0 = 2a(x + x_0 – 10) $$ $$
Given that $$y_0^2 = 4a(x_0 – 5) $$, we can rewrite as:
$$ $$y y_0 = 2a(x + x_0 – 10) $$ $$
This is the general form of the tangent to the parabola $$y^2 = 4a(x – 5) $$.
I’ll continue with more questions in my next response. Let me know if you’d like me to focus on any specific section or question next.
Retry
B
continue
Edit
I’ll continue with the solutions for the remaining questions.
Question 8(b): Let A, B and C be the midpoints of the sides ST, RT and SR of the triangle △RST respectively, whose coordinates are R(2, -7), S(4, -5), T(6, 3). Find the coordinates of the point of intersection P of the lines RA and SB. Show that TC passes through the point P.
Step 1: Find the coordinates of the midpoints A, B, and C.
– Midpoint A of side ST:
$$ $$A = \left(\frac{S_x + T_x}{2}, \frac{S_y + T_y}{2}\right) = \left(\frac{4 + 6}{2}, \frac{-5 + 3}{2}\right) = (5, -1) $$ $$
– Midpoint B of side RT:
$$ $$B = \left(\frac{R_x + T_x}{2}, \frac{R_y + T_y}{2}\right) = \left(\frac{2 + 6}{2}, \frac{-7 + 3}{2}\right) = (4, -2) $$ $$
– Midpoint C of side SR:
$$ $$C = \left(\frac{S_x + R_x}{2}, \frac{S_y + R_y}{2}\right) = \left(\frac{4 + 2}{2}, \frac{-5 + (-7)}{2}\right) = (3, -6) $$ $$
Step 2: Find the equation of line RA.
Line RA passes through R(2, -7) and A(5, -1).
The slope of RA is:
$$ $$m_{RA} = \frac{-1 – (-7)}{5 – 2} = \frac{6}{3} = 2 $$ $$
Equation of line RA using point-slope form:
$$ $$y – (-7) = 2(x – 2) $$ $$
$$ $$y + 7 = 2x – 4 $$ $$
$$ $$y = 2x – 11 $$ $$
Step 3: Find the equation of line SB.
Line SB passes through S(4, -5) and B(4, -2).
The slope of SB is:
$$ $$m_{SB} = \frac{-2 – (-5)}{4 – 4} = \frac{3}{0} $$ $$
Since the slope is undefined, the equation of line SB is:
$$ $$x = 4 $$ $$
Step 4: Find the point of intersection P of lines RA and SB.
From the equation of SB, we know that the x-coordinate of P is 4.
Substituting x = 4 into the equation of RA:
$$ $$y = 2(4) – 11 = 8 – 11 = -3 $$ $$
Therefore, the point of intersection P is (4, -3).
Step 5: Check if TC passes through point P.
Line TC passes through T(6, 3) and C(3, -6).
The slope of TC is:
$$ $$m_{TC} = \frac{-6 – 3}{3 – 6} = \frac{-9}{-3} = 3 $$ $$
Equation of line TC using point-slope form:
$$ $$y – 3 = 3(x – 6) $$ $$
$$ $$y – 3 = 3x – 18 $$ $$
$$ $$y = 3x – 15 $$ $$
To check if P(4, -3) lies on this line, substitute:
$$ $$-3 \stackrel{?}{=} 3(4) – 15 = 12 – 15 = -3 $$ $$
Since the equation is satisfied, P(4, -3) lies on line TC, which means TC passes through point P.
Question 8(c): Calculate the slope of the line parallel to the line joining the points (3,7) and (-2, -5).
To find the slope of a line joining two points $$(x_1, y_1) $$ and $$(x_2, y_2) $$, we use the formula:
$$ $$m = \frac{y_2 – y_1}{x_2 – x_1} $$ $$
For the line joining the points $$(3,7) $$ and $$(-2, -5) $$:
$$ $$m = \frac{-5 – 7}{-2 – 3} = \frac{-12}{-5} = \frac{12}{5} = 2.4 $$ $$
Since a parallel line has the same slope, the slope of the line parallel to the given line is also $$\frac{12}{5} $$ or $$2.4 $$.
SECTION C: STATISTICS
Question 9(a): A biased coin has the probability for a head 23\frac{2}{3} 32. The coin is tossed six times. Calculate the probability that:
Given:
– The probability of getting a head is $$p = \frac{2}{3} $$
– The probability of getting a tail is $$q = 1 – p = 1 – \frac{2}{3} = \frac{1}{3} $$
– The coin is tossed 6 times
We’ll use the binomial probability formula:
$$ $$P(X = k) = \binom{n}{k} p^k q^{n-k} $$ $$
(i) All six trials result in a tail:
This means getting 0 heads and 6 tails.
$$ $$P(X = 0) = \binom{6}{0} \left(\frac{2}{3}\right)^0 \left(\frac{1}{3}\right)^6 = 1 \times 1 \times \left(\frac{1}{3}\right)^6 = \frac{1}{729} $$ $$
(ii) None of the six results in a tail:
This means getting 6 heads and 0 tails.
$$ $$P(X = 6) = \binom{6}{6} \left(\frac{2}{3}\right)^6 \left(\frac{1}{3}\right)^0 = 1 \times \left(\frac{2}{3}\right)^6 \times 1 = \frac{64}{729} $$ $$
(iii) At least three heads appear in the six trials:
This means $$X \geq 3 $$, or $$P(X=3) + P(X=4) + P(X=5) + P(X=6) $$
$$ $$P(X \geq 3) = \sum_{k=3}^{6} \binom{6}{k} \left(\frac{2}{3}\right)^k \left(\frac{1}{3}\right)^{6-k} $$ $$
$$ $$P(X = 3) = \binom{6}{3} \left(\frac{2}{3}\right)^3 \left(\frac{1}{3}\right)^3 = 20 \times \frac{8}{27} \times \frac{1}{27} = \frac{160}{729} $$ $$
$$ $$P(X = 4) = \binom{6}{4} \left(\frac{2}{3}\right)^4 \left(\frac{1}{3}\right)^2 = 15 \times \frac{16}{81} \times \frac{1}{9} = \frac{240}{729} $$ $$
$$ $$P(X = 5) = \binom{6}{5} \left(\frac{2}{3}\right)^5 \left(\frac{1}{3}\right)^1 = 6 \times \frac{32}{243} \times \frac{1}{3} = \frac{192}{729} $$ $$
$$ $$P(X = 6) = \frac{64}{729} $$ $$ (from part ii)
$$ $$P(X \geq 3) = \frac{160}{729} + \frac{240}{729} + \frac{192}{729} + \frac{64}{729} = \frac{656}{729} $$ $$
(iv) Not more than three tails appear:
This means at least 3 heads (same as part iii).
$$ $$P(\text{tails} \leq 3) = P(\text{heads} \geq 3) = \frac{656}{729} $$ $$
(v) The first two results in a head, and the remaining four in a tail:
This is a specific sequence: HH(4 tails)
$$ $$P(HHTTTT) = \left(\frac{2}{3}\right)^2 \times \left(\frac{1}{3}\right)^4 = \frac{4}{9} \times \frac{1}{81} = \frac{4}{729} $$ $$
(vi) The heads and tails appear alternatively:
There are two possible alternating sequences: HTHTHTHT… or THTHTHTH…
First sequence: HTHTHT
Second sequence: THTHTH
$$ $$P(\text{alternating}) = P(HTHTHT) + P(THTHTH) $$ $$
$$ $$P(HTHTHT) = \left(\frac{2}{3}\right)^3 \times \left(\frac{1}{3}\right)^3 = \frac{8}{27} \times \frac{1}{27} = \frac{8}{729} $$ $$
$$ $$P(THTHTH) = \left(\frac{1}{3}\right)^3 \times \left(\frac{2}{3}\right)^3 = \frac{1}{27} \times \frac{8}{27} = \frac{8}{729} $$ $$
$$ $$P(\text{alternating}) = \frac{8}{729} + \frac{8}{729} = \frac{16}{729} $$ $$
Question 9(b): Find the Harmonic and Geometric means of 3, 5 and 7.
Given numbers: 3, 5, and 7
(1) Harmonic Mean (H):
$$ $$H = \frac{n}{\frac{1}{x_1} + \frac{1}{x_2} + … + \frac{1}{x_n}} $$ $$
For our values:
$$ $$H = \frac{3}{\frac{1}{3} + \frac{1}{5} + \frac{1}{7}} $$ $$
$$ $$H = \frac{3}{\frac{35 + 21 + 15}{105}} $$ $$
$$ $$H = \frac{3}{\frac{71}{105}} $$ $$
$$ $$H = 3 \times \frac{105}{71} $$ $$
$$ $$H = \frac{315}{71} \approx 4.44 $$ $$
(2) Geometric Mean (G):
$$ $$G = \sqrt[n]{x_1 \times x_2 \times … \times x_n} $$ $$
For our values:
$$ $$G = \sqrt[3]{3 \times 5 \times 7} $$ $$
$$ $$G = \sqrt[3]{105} $$ $$
$$ $$G \approx 4.73 $$ $$
Therefore:
– Harmonic Mean = $$\frac{315}{71} \approx 4.44 $$
– Geometric Mean = $$\sqrt[3]{105} \approx 4.73 $$
Question 10(a): Two dice are tossed once, what is the probability that the:
When two dice are tossed, there are 36 possible outcomes (6×6 = 36), each with equal probability.
(i) Sum is less than 7:
Favorable outcomes: (1,1), (1,2), (1,3), (1,4), (1,5), (2,1), (2,2), (2,3), (2,4), (3,1), (3,2), (3,3), (4,1), (4,2), (5,1)
Total favorable outcomes = 15
$$ $$P(\text{sum} < 7) = \frac{15}{36} = \frac{5}{12} $$ $$
(ii) Same number appeared:
Favorable outcomes: (1,1), (2,2), (3,3), (4,4), (5,5), (6,6)
Total favorable outcomes = 6
$$ $$P(\text{same number}) = \frac{6}{36} = \frac{1}{6} $$ $$
(iii) Difference is 2:
Favorable outcomes: (1,3), (2,4), (3,5), (4,6), (3,1), (4,2), (5,3), (6,4)
Total favorable outcomes = 8
$$ $$P(\text{difference} = 2) = \frac{8}{36} = \frac{2}{9} $$ $$
(iv) Product is greater than 8:
Favorable outcomes: (2,5), (2,6), (3,3), (3,4), (3,5), (3,6), (4,3), (4,4), (4,5), (4,6), (5,2), (5,3), (5,4), (5,5), (5,6), (6,2), (6,3), (6,4), (6,5), (6,6)
Total favorable outcomes = 20
$$ $$P(\text{product} > 8) = \frac{20}{36} = \frac{5}{9} $$ $$
(v) First number is even:
Favorable outcomes: all outcomes with first die showing 2, 4, or 6
Total favorable outcomes = 18
$$ $$P(\text{first number even}) = \frac{18}{36} = \frac{1}{2} $$ $$
Question 10(b): If the diameters of ball bearings are normally distributed with mean 60.5cm and standard deviation 2.5cm, determine the probability of the ball bearing with diameter between 58.5cm and 63.5cm.
Given:
– Mean (μ) = 60.5 cm
– Standard deviation (σ) = 2.5 cm
– We want to find P(58.5 < X < 63.5)
Step 1: Standardize to Z-scores.
For X = 58.5:
$$ $$Z_1 = \frac{58.5 – 60.5}{2.5} = \frac{-2}{2.5} = -0.8 $$ $$
For X = 63.5:
$$ $$Z_2 = \frac{63.5 – 60.5}{2.5} = \frac{3}{2.5} = 1.2 $$ $$
Step 2: Find the probability using the standard normal distribution table or formula.
$$ $$P(58.5 < X < 63.5) = P(-0.8 < Z < 1.2) $$ $$
$$ $$P(58.5 < X < 63.5) = P(Z < 1.2) – P(Z < -0.8) $$ $$
$$ $$P(58.5 < X < 63.5) = \Phi(1.2) – \Phi(-0.8) $$ $$
$$ $$P(58.5 < X < 63.5) = \Phi(1.2) – (1 – \Phi(0.8)) $$ $$
$$ $$P(58.5 < X < 63.5) = \Phi(1.2) + \Phi(0.8) – 1 $$ $$
From standard normal tables:
– $$\Phi(1.2) \approx 0.8849 $$
– $$\Phi(0.8) \approx 0.7881 $$
$$ $$P(58.5 < X < 63.5) = 0.8849 + 0.7881 – 1 = 0.673 $$ $$
Therefore, the probability of a ball bearing having a diameter between 58.5 cm and 63.5 cm is approximately 0.673 or 67.3%.
Question 11(a): If f(x)=mx2f(x) = mx^2 f(x)=mx2 for 0≤x≤20 \leq x \leq 2 0≤x≤2 is a probability density function for a continuous random variable x, calculate:
For a function to be a valid PDF:
1. $$f(x) \geq 0 $$ for all $$x $$ in the domain
2. $$\int_{-\infty}^{\infty} f(x) dx = 1 $$
(i) The value of m:
Using condition 2 and our domain $$0 \leq x \leq 2 $$:
$$ $$\int_{0}^{2} mx^2 dx = 1 $$ $$
$$ $$m \left[ \frac{x^3}{3} \right]_{0}^{2} = 1 $$ $$
$$ $$m \left( \frac{8}{3} – 0 \right) = 1 $$ $$
$$ $$m = \frac{3}{8} $$ $$
(ii) P(0≤x≤1):
$$ $$P(0 \leq x \leq 1) = \int_{0}^{1} \frac{3}{8}x^2 dx $$ $$
$$ $$P(0 \leq x \leq 1) = \frac{3}{8} \left[ \frac{x^3}{3} \right]_{0}^{1} $$ $$
$$ $$P(0 \leq x \leq 1) = \frac{3}{8} \left( \frac{1}{3} – 0 \right) $$ $$
$$ $$P(0 \leq x \leq 1) = \frac{3}{8} \times \frac{1}{3} = \frac{1}{8} $$ $$
(iii) The mean:
$$ $$\mu = \int_{0}^{2} x \cdot f(x) dx = \int_{0}^{2} x \cdot \frac{3}{8}x^2 dx = \frac{3}{8} \int_{0}^{2} x^3 dx $$ $$
$$ $$\mu = \frac{3}{8} \left[ \frac{x^4}{4} \right]_{0}^{2} $$ $$
$$ $$\mu = \frac{3}{8} \left( \frac{16}{4} – 0 \right) $$ $$
$$ $$\mu = \frac{3}{8} \times 4 = \frac{12}{8} = \frac{3}{2} $$ $$
(iv) The variance:
$$ $$\sigma^2 = \int_{0}^{2} (x-\mu)^2 \cdot f(x) dx $$ $$
$$ $$\sigma^2 = \int_{0}^{2} \left(x-\frac{3}{2}\right)^2 \cdot \frac{3}{8}x^2 dx $$ $$
An alternative approach is to use the formula:
$$ $$\sigma^2 = E(X^2) – \mu^2 $$ $$
First, find $$E(X^2) $$:
$$ $$E(X^2) = \int_{0}^{2} x^2 \cdot f(x) dx = \int_{0}^{2} x^2 \cdot \frac{3}{8}x^2 dx = \frac{3}{8} \int_{0}^{2} x^4 dx $$ $$
$$ $$E(X^2) = \frac{3}{8} \left[ \frac{x^5}{5} \right]_{0}^{2} $$ $$
$$ $$E(X^2) = \frac{3}{8} \left( \frac{32}{5} – 0 \right) $$ $$
$$ $$E(X^2) = \frac{3}{8} \times \frac{32}{5} = \frac{96}{40} = \frac{12}{5} $$ $$
Now calculate the variance:
$$ $$\sigma^2 = E(X^2) – \mu^2 = \frac{12}{5} – \left(\frac{3}{2}\right)^2 = \frac{12}{5} – \frac{9}{4} $$ $$
$$ $$\sigma^2 = \frac{48 – 45}{20} = \frac{3}{20} = 0.15 $$ $$
Question 11(b): On an average, two accidents occur every day on a busy highway and it is known that the number of accidents per day follows poison law. Find the probability that on a certain day:
Given:
– Mean number of accidents per day (λ) = 2
– The number of accidents follows a Poisson distribution
The Poisson probability mass function is:
$$ $$P(X = k) = \frac{e^{-\lambda} \lambda^k}{k!} $$ $$
(i) There are 2 accidents:
$$ $$P(X = 2) = \frac{e^{-2} \times 2^2}{2!} = \frac{e^{-2} \times 4}{2} = 2e^{-2} = 2 \times 0.1353 = 0.2706 $$ $$
(ii) There is no accident:
$$ $$P(X = 0) = \frac{e^{-2} \times 2^0}{0!} = e^{-2} = 0.1353 $$ $$
(iii) Find the least value, n such that the probability for n accidents on a particular day is <0.1.
We need to find the smallest n such that P(X = n) < 0.1:
$$ $$P(X = n) = \frac{e^{-2} \times 2^n}{n!} < 0.1 $$ $$
Let’s check for different values of n:
For n = 5:
$$ $$P(X = 5) = \frac{e^{-2} \times 2^5}{5!} = \frac{e^{-2} \times 32}{120} = \frac{32 \times 0.1353}{120} = \frac{4.3296}{120} = 0.0361 $$ $$
For n = 4:
$$ $$P(X = 4) = \frac{e^{-2} \times 2^4}{4!} = \frac{e^{-2} \times 16}{24} = \frac{16 \times 0.1353}{24} = \frac{2.1648}{24} = 0.0902 $$ $$
Since P(X = 4) < 0.1 and P(X = 3) ≈ 0.1804 > 0.1, the least value of n is 4.
[Approximately take e^{-2} = 1/7]